0
$\begingroup$

I don't know how one can tackle the following kind of question, so any hint is welcome. I formulate a precise question in order to fix ideas, but it is to be considered as an example out of a more general class.

Example: Can one embed Petersen's graph in $\mathbb R^4$ in such a way that all edges are mapped to segments (of not necessarily equal lenght) and each pair of adjacent segments forms an angle of $2\pi/3$ ?

The question can be generalized in the following way, to give a wider class of questions:

  • instead of Petersen's graph, consider a given $k$-regular graph (we had $k=3$ above)
  • instead of $\mathbb R^4$ consider $\mathbb R^n, n>k$, or embeddings into the round sphere $S^n$ where the edges of the graph are mapped into geodesics
  • ask that the angles formed between any pair of adjacent edges are equal to the angle $VOV'$ made at which two vertices $V,V'$ of a regular euclidean $(k-1)$-simplex are seen from its barycenter $O$

I think that $1$-skeletons of spherical regular polytopes give such kind of graphs. I am interested in any partial answer which introduces essentially more ingredients than just constructions based on symmetry groups, or in obstructions which show negative answers.

$\endgroup$

1 Answer 1

4
$\begingroup$

Assume $\Gamma$ is a 3-regular graph in $\mathbb{R}^n$ and the angles at each vertex are $\tfrac23\pi$. Then it is easy to see that the distance to the origin can not have a local maximum on $\Gamma$.

Therefore $\Gamma$ has to be infinite; in particular it can not be Petersen's graph.

$\endgroup$
2
  • $\begingroup$ Thanks, I didn't notice this criterion. If there will not be more answers (for example on $S^n$ or for other $k$) within 2 weeks, I will accept this one. $\endgroup$
    – Mircea
    Jul 9, 2012 at 14:33
  • 2
    $\begingroup$ Yet an other reason: the Petersen's graph has cycles of length 5; any 5-gon in $\mathbb R^k$ has sum of angles at most $3\cdot\pi$, so at least one of the angles has to be $\le \tfrac35\cdot\pi<\tfrac23\cdot\pi$. $\endgroup$ Jul 9, 2012 at 17:11

Your Answer

By clicking “Post Your Answer”, you agree to our terms of service and acknowledge you have read our privacy policy.

Not the answer you're looking for? Browse other questions tagged or ask your own question.